Questions tagged [computational-number-theory]

Computational Number Theory is for explicit calculations or algorithms involving anything of interest to number theorists.

159 questions with no upvoted or accepted answers
Filter by
Sorted by
Tagged with
18 votes
0 answers
899 views

Computation of low weight Siegel modular forms

We have these huge tables of elliptic curves, which were generated by computing modular forms of weight $2$ and level $\Gamma_0(N)$ as N increased. For abelian surfaces over $\mathbb{Q}$ we have very ...
Kevin Buzzard's user avatar
15 votes
0 answers
337 views

Do primes of the form $4k+1$ ever lead the greatest prime factor race?

Analogous to Chebyshev's race between primes, I examined the race between primes in the greatest prime factors, GPF, of natural numbers. Similar to the regular prime race, in the GPF race, the ...
Nilotpal Kanti Sinha's user avatar
13 votes
0 answers
1k views

Why am I unable to find primes of the form $(9n)!+n!+1$?

See also Math StackExchange: Is there a prime of the form $(9n)!+n!+1$? Recently, user Peter from Math StackExchange asked for a prime of the form $(9n)!+n!+1$ (where $n$ is some natural number). ...
Maximilian Janisch's user avatar
12 votes
0 answers
1k views

Euler's totient function and Riemann hypothesis

I am looking for an upper-bound of the Euler's totient function $\varphi$ which would be equivalent to the Riemann hypothesis (RH). There is the following Nicolas' criterion about primorial numbers $...
Sebastien Palcoux's user avatar
11 votes
0 answers
765 views

Borderline Collatz-like problems

The usual Collatz map is $C:n \mapsto n/2$ if $n$ even, $(3n+1)/2$ if $n$ odd. Let $f^{\circ (r+1)}:=f \circ f^{\circ r}$. We suspect that for every fixed $n>0$, the sequence $C^{\circ r}(n)$ ...
Sebastien Palcoux's user avatar
11 votes
0 answers
826 views

Points of bounded height in a number field

Let $K$ be a number field of absolute degree $d$, let $B$ be a positive real number, and write $S(K, B) = \{x \in K : H(x) \leq B\}$. Here $H$ is the absolute multiplicative height of an algebraic ...
Xander Faber's user avatar
  • 1,169
8 votes
0 answers
232 views

Hilbert 10th problem for genus 2 equations

Hilbert 10th problem, while undecidable in general, remains open for 2-variable equations: we do not know if there is an algorithm that, for polynomial $P(x,y)$ with integer coefficients, decides ...
Bogdan Grechuk's user avatar
8 votes
1 answer
471 views

How large can the dimension of a 'Span of powers of a finite field basis' be?

Let $p$ be a prime. For finite field $\mathbb{F}_{p^k}$ and $d\in\mathbb{Z}^+$, I am considering the following quantity, where we interpret the field $\mathbb{F}_{p^k}$ also as a $\mathbb{F}_p$-vector ...
actcon's user avatar
  • 89
8 votes
0 answers
371 views

Computing motivic Galois group

Suppose I have a motive $M$ over $\mathbb{Q}$, and can compute the Euler factor of the associated $L$-function for any good prime $p$. How can I compute the Zariski closure of the image of the Galois ...
Watson Ladd's user avatar
  • 2,419
8 votes
0 answers
235 views

Computing the Moebius function $\mu$

Is it known whether computing $\mu(n)$ for a given integer $n$ is as hard as factorization?
H A Helfgott's user avatar
  • 19.4k
7 votes
0 answers
272 views

Are there infinitely many zeroes of $\sum_{r = 1}^{n-1} \mu(r)\gcd(n,r) $?

Let $\mu(n)$ be the Möbius function and $S(x)$ be the number of positive integers $n \le x$ such that $$ \sum_{r = 1}^{n-1} \mu(r)\gcd(n,r) = 0 $$ My experimental data for $n \le 6 \times 10^5 $...
Nilotpal Kanti Sinha's user avatar
7 votes
0 answers
607 views

Proving Richardson's theorem for constants

(I asked this a little over 3 months ago on math.SE, and when I initially re-asked here, no one had responded there. $\:$ After I re-asked here, Eric Towers responded there, since I had forgotten to ...
user avatar
7 votes
0 answers
293 views

On the ratio of Gilbreath sequences

Definitions: let $n \in \mathbb{N}_{>0} \cup \{ \infty \}$ and let $E_n$ be the set of sequences $(d_i)_{i=1}^n$ such that $d_1=1$, $d_i$ is an even integer (for $i > 1$) and $0<d_i \le i$. ...
Sebastien Palcoux's user avatar
6 votes
0 answers
191 views

Newton type method for finite fields?

I have a polynomial $p(x)$ in $\mathbb{Z}/q\mathbb{Z}$ that is easy to compute for any $x$ but has an absurdly large degree $d > 2^{256}$. I know for a fact that it has a zero and I would like to ...
mtheorylord's user avatar
6 votes
0 answers
109 views

Equivalence of primes based on the partition of their Pisano periods

The period of Fibonacci numbers modulo $m$ is called Pisano period and its length is denoted as $\pi(m)$. Define the Pisano partition of $m$ as the set partition of the indices $\{0,1,\dotsc,\pi(m)-1\}...
Max Alekseyev's user avatar
6 votes
0 answers
159 views

Certificate for computation of ideal class group

Is there a known way of producing a certificate that can be used to more quickly verify that an ideal class group of a number field was computed correctly? More formally, I would like to know if there'...
Daniel Hast's user avatar
  • 1,806
6 votes
0 answers
90 views

Computing all eta quotients of given weight and level

I have written a rather naive program for finding all holomorphic eta quotients of given weight and level (and varying character). When the level has few divisors it is very fast, but incredibly slow ...
Henri Cohen's user avatar
  • 11.8k
6 votes
0 answers
288 views

Effective bounds for Fermat's Last Theorem

Suppose $n>2$. By Fermat's Last Theorem, we know that $a^{n}+b^{n}=c^{n}$ has no non-trivial solutions. Can we quantify it more? More specifically, given $a,b,c,n\in\mathbb{N}$ with $n>2$ and $...
Gorav Jindal's user avatar
6 votes
0 answers
253 views

Brief history of primality testing theory after 2002?

Its clear that there is about 15 years (2004-2019) after the publication of AKS primality testing in 2002 and its modifications in 2003-2004. AS result, is there any development happened in this ...
Ramez Hindi's user avatar
6 votes
0 answers
427 views

Are there always at least *five* divisions?

@JosephO'Rourke asked a question about a Collatz like function related to primes: $f(n) = \begin{cases} n^2 & \text{if} \;n \;\text{is prime} \\ \lfloor n/2 \rfloor & \text{if} \;n \;\text{...
Mirko's user avatar
  • 1,345
5 votes
0 answers
175 views

Empirical bounds on $\left|\frac{\zeta'(1+it)}{\zeta(1+it)}\right|$

It is reasonable to expect that $$\left|\frac{\zeta'(1+it)}{\zeta(1+it)}\right| < 2 \log \log t$$ for all $t\geq 4$ (say): a somewhat stronger bound is known for $t\geq 10^{165}$ or so (Theorem 5 ...
H A Helfgott's user avatar
  • 19.4k
5 votes
0 answers
304 views

A question on infinite arithmetic progressions

I was working on a problem that consisted of deciding if the language a finite automaton (the alphabet of which is $\{0,1\}$ and the words accepted are binary encoded positive integers) contains an ...
Irmak Sağlam's user avatar
5 votes
0 answers
197 views

Ramsey Numbers for Integers

Erdos defined $f(n)$ to be the minimum $r$ such that there is an $r$-coloring of the positive integers less than $n$, wherein $n$ cannot be written as the sum of distinct monochromatic integers. ...
Yoozer2021's user avatar
5 votes
0 answers
337 views

About a diophantine equation from group theory

Is there any set of odd primes $\{p_1, p_2,..., p_k\}$ and natural numbers $a_1,..., a_k$ such that the following equation satisfied: $${p_1^{2a_1+1}+1 \over p_1+1}\times ....\times {p_k^{2a_k+1}+1 \...
BHZ's user avatar
  • 1,168
5 votes
0 answers
173 views

Finding a presentation matrix with low dimension

Let $R=\mathbb Z[t^{\pm}]$ and $M$ a finitely generated $R$-module. With $A$ a presentation matrix, i.e we have the following exact sequence (usually I'm working with the case where $A$ is an square ...
bruno mazorra's user avatar
5 votes
0 answers
195 views

Is there some computational evidence of the $pq$ analog of Serre's conjecture?

The $pq$ analog of Serre's conjecture (see "Mod pq Galois representations and Serre's Conjecture"- Khare, Kiming) states that if $\bar{\rho}_1:G_{\mathbb{Q}}\rightarrow \text{GL}_2(\mathbb{F}_p)$ is a ...
user avatar
5 votes
0 answers
125 views

Anyone got two Galois reps to compare?

I've got a new criterion for comparing Galois reps which are four dimensional if we know the kernel of the residual representation mod $5$ (or any large odd prime) and the Sato-Tate groups (should be ...
Watson Ladd's user avatar
  • 2,419
5 votes
0 answers
711 views

Primitive element for a number field, and ramification

Let $K=\mathbb Q(\theta)$ be a number field with integral primitive element $\theta$, and let $f(x)$ be the minimal polynomial of $\theta$. Let $p$ be a rational prime. It's well known that if $p$ ...
352506's user avatar
  • 1,001
5 votes
0 answers
153 views

On factorization algorithms for $\mathcal{O}[x]$

We know that $\mathsf{LLL}$ algorithm provides factorization procedure that runs in poly time for polynomials in $\Bbb Z[x]$ that are primitive. What other rings $\mathcal{O}$ can we use instead of $\...
user avatar
5 votes
0 answers
286 views

Lower bound for p-adic distance between roots

Let $f$ be a formal power series with coefficients in the ring of integers of a finite extension of ${\mathbb Q}_p$. Is there a simple algorithm to compute a positive lower bound for $|\alpha - \beta|...
Jesse Elliott's user avatar
5 votes
0 answers
228 views

Example of level one cuspidal Hecke Algebra T_k^0 such that p divides its index in its normalization, and p≥k-1?

The question is strongly focused on computations concerning modular forms and Hecke algebras. It is already in the title, but I will repeat it, adding a few details. Let $S_k$ be the complex vector ...
Tommaso Centeleghe's user avatar
4 votes
0 answers
741 views

One of the numbers $\zeta(5), \zeta(7), \zeta(9), \zeta(11)$ is irrational

I am reading an interesting paper One of the numbers ζ(5), ζ(7), ζ(9), ζ(11) is irrational by Zudilin. We fix odd numbers $q$ and $r$, $q\geq r+4$ and a tuple $\eta_0,\eta_1,...,\eta_q$ of positive ...
Max's user avatar
  • 1
4 votes
0 answers
42 views

Computing preimage of element under norm map of quadratic extension of $2$-adic fields

Let $F$ be a $2$-adic field, i.e. a finite extension of the $2$-adic numbers $\mathbb{Q}_2$. Suppose that I have a quadratic extension $E = F(\sqrt{d})$ of $F$. Given a unit $\alpha \in \mathcal{O}_F^\...
Sebastian Monnet's user avatar
4 votes
0 answers
296 views

Are there infinitely many simple integral fusion rings of rank $4$?

$\DeclareMathOperator\ch{ch}$$\DeclareMathOperator\FPdim{FPdim}$We refer to [EGNO15, Chapter 3] for the notion of fusion ring and basic results. The type of a fusion ring $R$ is the list $(\FPdim(b_i)...
Sebastien Palcoux's user avatar
4 votes
0 answers
122 views

Road map for learning about the computational/general theory of modular curves/isogenies of abelian varieties for cryptography

I am a graduate math/crypto student. So I've had some free time last year and I heard about elliptic curves in cryptography and how a resilient cryptosystem got demolished by a spectacular attack ...
Rayane B.'s user avatar
4 votes
0 answers
184 views

What are the modularity conjectures for Artin motives?

Classically, singular cohomology is an important tool for studying topological spaces, in particular, complex varieties. In the mid-twentieth century it was realized that there are many analogues of ...
David Schwein's user avatar
4 votes
0 answers
123 views

Lattice reduction of basis with non-integer coefficients

Suppose I have an ordered basis $\{b_1, \dots, b_n\}$ of a lattice in $\mathbb{R}^n$, but I do not assume that $b_i \in \mathbb{Z}^n$ for all $1 \leq i \leq n$. I would like to perform lattice ...
george's user avatar
  • 554
4 votes
0 answers
101 views

Questions in number theory related to $NC$ and $P$-completeness

Given $a,b\in\mathbb N$ find $\operatorname{GCD}(a,b)$. Given $a,b,c\in\mathbb N$ find $x,y\in\mathbb Z$ such that $ax+by=c$. Euclidean algorithm solves both. My question is if either 1 or 2 is in ...
Turbo's user avatar
  • 13.7k
4 votes
1 answer
282 views

Find $\mathbb{Z}$-basis of module over Dedekind domain provided its pseudobasis

Let $K$ be number field of degree $d$. Suppose we are given module $ \mathcal{M}$ in form: \begin{equation}\label{key} \mathcal{M} = v_1 \cdot \mathfrak{a}_1 \oplus v_2 \cdot \mathfrak{a}_2 \...
Александр Каренин's user avatar
4 votes
0 answers
99 views

Reconstructing coefficients of an elliptic curve L-series from the modular form divisor

Let $E$ be an unknown elliptic curve over $\mathbb{Q}$. Let $L(E, s) = \sum_{n=1}^{\infty} \frac{a_n}{n^s}$ be the L-function of $E$ and write $f(q) = \sum_{n=1}^{\infty} a_n q^n$. I'm in a setting ...
Andreas Holmstrom's user avatar
4 votes
0 answers
142 views

Riemann-Siegel formula for Dirichlet characters

After unearthing and giving a proof of what is now known as the Riemann--Siegel formula for the Riemann zeta function enabling the computation of $\zeta(1/2+iT)$ in time $O(T^{1/2})$, in 1943 Siegel ...
Henri Cohen's user avatar
  • 11.8k
4 votes
0 answers
210 views

What is the complexity class of this problem without Cramer's conjecture?

The problem 'Given $0<a<b$ is there a prime in the interval $[a,b]$?' is in $\mathsf{NP}$. If we assume Cramer's conjecture the problem is in $\mathsf{P}$ since if $b-a>(\log a)^{2+\epsilon}$ ...
Turbo's user avatar
  • 13.7k
4 votes
0 answers
429 views

The irrational numbers α such that n odd and m=⌊nα⌋ odd implies ⌊mα⌋ odd

This post is the analogous of that one (about $\sqrt{2}$) but with a much stronger expectation here. We observed, and then this comment of Lucia proved, that for $\phi$ the golden ratio, if $n$ ...
Sebastien Palcoux's user avatar
4 votes
0 answers
186 views

How to find a CM point with the image in the elliptic curve under modular parametrization given

everyone! Let $E:y^2+y=x^3-61$ be the minimal model of the elliptic curve 243b. How can I find the CM point $\tau$ in $X_0(243)$ such that $\tau$ maps to the point $(3\sqrt[3]{3},4)$ under the modular ...
yhb's user avatar
  • 338
4 votes
0 answers
118 views

Finding short linear combinations in abelian groups

Let $M$ be a finitely generated abelian group. Assume we are given a presentation of $M$, that is \begin{equation*} M = \frac{\bigoplus_{i=1}^r \mathbf{Z}g_i}{\sum_{j=1}^s \mathbf{Z} r_j} \end{...
François Brunault's user avatar
4 votes
0 answers
274 views

Parity of number of primes

In https://arxiv.org/abs/1009.3956 is it shown there is a $c>0$ such that $\pi(x)\bmod2$ can be computed in $o(x^{\frac12})$ time (more precisely number of primes $\bmod 2$ for an interval of ...
Turbo's user avatar
  • 13.7k
4 votes
0 answers
176 views

Computing the density of a set of multiples

Erdős and his coauthors often wrote about problems relating to the densities of sets of multiples. I have a computational question about the same topic. I have a finite* set $A=a_1<\cdots<a_r$ ...
Charles's user avatar
  • 8,984
4 votes
0 answers
203 views

Monte Carlo variant of Hilbert's Tenth Problem

Let $k \in \mathbb{N}$. Given an algorithm $\mathcal{A}$ which takes as argument a polynomial $P \in \mathbb{Z}[x_1,\dots,x_k]$ and either returns true or false, we say that $\mathcal{A}$ works for $P$...
Stefan Kohl's user avatar
  • 19.5k
4 votes
0 answers
245 views

Algorithm/denominators of elements of a rational affine space

I hope it's not a trivial question... Suppose I have a finite dimensional vector space $V$ over $\mathbb{Q}$ with a distinguished basis (in my case it's the $k$th graded piece of the free associative ...
Adrien's user avatar
  • 8,244
4 votes
0 answers
159 views

Range of the least witness function

Let W(n) be a function from the positive odd composite numbers to the least positive b such that n is not a b-strong pseudoprime. W(n) exists for all numbers in its domain and its range is unbounded. ...
Charles's user avatar
  • 8,984